Đến nội dung

quanghung86 nội dung

Có 489 mục bởi quanghung86 (Tìm giới hạn từ 02-05-2020)



Sắp theo                Sắp xếp  

#667272 Đề Thi VMO năm 2017

Đã gửi bởi quanghung86 on 06-01-2017 - 14:12 trong Thi HSG Quốc gia và Quốc tế

Figure4239.png

 

Chi tiết hơn nhé. $\angle EGF=\angle BGE+\angle CGF-\angle EGF=360^\circ-2\angle BAC-(180^\circ-2\angle BAC)=180^\circ$ nên $E,G,F$ thẳng hàng. Từ đó $\angle ABK+\angle ACK=\angle AGE+\angle AGF=180^\circ$ nên $K$ thuộc $(O)$. Dễ thấy $G$ là điểm Miquel nên hai tam giác $GBA$ và $GKC$ đồng dạng, lại có $GO$ là phân giác $\angle BGC$ nên $GO$ là phân giác $\angle AGK$. Từ đó kết hợp $OA=OK$ thì $AOKG$ nội tiếp. Sử dụng trục đẳng phương dễ thấy $AK,OG,BC$ đồng quy.




#667279 Đề Thi VMO năm 2017

Đã gửi bởi quanghung86 on 06-01-2017 - 14:44 trong Thi HSG Quốc gia và Quốc tế

Có thể xem thêm lời giải và bình luận hai đề hình ở đây

 

http://analgeomatica...i-hinh-thi.html




#667266 Đề Thi VMO năm 2017

Đã gửi bởi quanghung86 on 06-01-2017 - 13:12 trong Thi HSG Quốc gia và Quốc tế

Đề hình này có lẽ hay ở ý b). Ý b) chỉ cần $(O),(I)$ là hai đường tròn cố định qua $B,C$ cố định là được. $D$ cố định và $G$ di chuyển. Ta làm như sau

 

Untitled.png

 

Áp dụng Pascal dễ chỉ ra $M,N$ đi qua giao tiếp tuyến tại $B,C$ là $J$ của $(I)$ cố định. Gọi $JD$ cắt $(I)$ tại $X$ thì $X$ cố định và tứ giác $BCDX$ điều hòa nên $G(BC,DX)=-1$. Gọi $MN$ cắt $BC$ tại $T$ thì $G(BC,DT)=-1$ từ đó $GX$ đi qua $T$. Nên TX.TG=TB.TC=TP.TQ suy ra $(GPQ)$ đi qua $X$ cố định. Gọi $(GPQ)$ cắt $DX$ tại $Y$ thì $JX.JY=JP.JQ$ bằng phương tích của $J$ đối với $(O)$ nên $Y$ cố định. Suy ra $(GPQ)$ đi qua $X,Y$ cố định.




#667079 Đề Thi VMO năm 2017

Đã gửi bởi quanghung86 on 05-01-2017 - 13:43 trong Thi HSG Quốc gia và Quốc tế

Hy vọng bài viết này giúp ích giải câu b)

 

http://analgeomatica...-dien-aops.html




#668627 VMF's Marathon Hình học Olympic

Đã gửi bởi quanghung86 on 17-01-2017 - 00:58 trong Hình học

vietdohoangtk7nqd cần đề nghị bài tiếp nhưng vì lâu nên để bạn ấy đề nghị sau, để topic không bị gián đoán mình xin đề nghị bài tiếp

 

Bài toán 126 (AoPS). Cho tam giác $ABC$ có đường tròn nội tiếp $(I)$ tiếp xúc $BC,CA,AB$ tại $D,E,F$. $P$ nằm trên đường thẳng $OI$ của tam giác $ABC$. $X,Y,Z$ đối xứng $P$ qua $IA,IB,IC$. Chứng minh rằng $DX,EY,FZ$ đồng quy.




#668330 VMF's Marathon Hình học Olympic

Đã gửi bởi quanghung86 on 14-01-2017 - 22:32 trong Hình học

Cám ơn khánh, thầy sẽ xem lại đề kỹ hơn, để topic tiếp tục, thầy đề nghị bài mới

Bài toán 123. Cho tứ giác $ABCD$ nội tiếp đường tròn $(O)$. $AB$ giao $CD$ tại $E$. $AD$ giao $BC$ tại $F$. Tiếp tuyến tại $A,C$ của $(O)$ cắt nhau tại $S$. Tiếp tuyến tại $B,D$ của $(O)$ cắt nhau tại $T$. Gọi $K,L$ là tâm ngoại tiếp tam giác $SBD$ và $TAC$. $M$ là trung điểm $EF$. Chứng minh rằng $OM\parallel KL$.



#667974 VMF's Marathon Hình học Olympic

Đã gửi bởi quanghung86 on 11-01-2017 - 11:01 trong Hình học

Mình đưa ra tổng quát bài toán 109 và lời giải của mình, nguồn gốc bài 109 là bài số 6 chọn đội Iran 2014 chứ không phải 2015

 

https://www.artofpro...h590555p3497371

 

Bài toán 109'. Cho tam giác $ABC$ nội tiếp đường tròn $(O)$ với tâm nội tiếp $I$ và tâm bàng tiếp góc $A$ là $J$. $P$ là trung điểm cung $BC$ chứa $A$ của $(O)$. $X$ thuộc $JP$. $E,F$ là hình chiếu của $X$ lên $IB,IC$. Chứng minh rằng $OP$ chia đôi $EF$.

 

Figure4302.png

 

Lời giải. Gọi $K,L$ là tâm bàng tiếp góc $B,C$ của $ABC$ thì $P$ là trung điểm $KL$. Gọi $XF,XE$ cắt $JC,JB$ tại $QR$. Dễ thấy do $XF\parallel JB$ và $XE\parallel JC$ nên $QR\parallel BC$. Từ đó $\frac{FB}{FK}=\frac{QJ}{QK}=\frac{RJ}{RL}=\frac{EC}{EL}$ nên theo bổ đề E.R.I.Q thì trung điểm của $KL,EF,BC$ thẳng hàng. Ta hoàn thành chứng minh.




#667969 VMF's Marathon Hình học Olympic

Đã gửi bởi quanghung86 on 11-01-2017 - 10:21 trong Hình học

Cám ơn Hoàng và Khánh, một lời giải khác cho bài toán 107 của thầy Hà ở đây, rất thú vị và ngắn gọn

 

https://www.artofpro...c6t48f6h1366405

 

Thầy giúp Hoàng đề nghị bài toán sau

 

Bài toán 111. Cho tam giác $ABC$ nội tiếp đường tròn $(O)$. Đường tròn tâm nội tiếp $I$. Đường tròn $(K)$ tiếp xúc $CA,AB$ tại $E,F$ và tiếp xúc trong $(O)$. $P$ là trung điểm cung $BC$ chứa $A$ của $(O)$. $PI$ cắt $BC$ tại $D$. $Q$ là trung điểm cung nhỏ $AP$ của $(O)$. $R$ là trung điểm cung nhỏ $EF$ của $(K)$. $M$ là trung điểm $ID$. Chứng minh rằng $MQ\parallel DR$.

 

Figure4301.png




#669146 VMF's Marathon Hình học Olympic

Đã gửi bởi quanghung86 on 20-01-2017 - 23:24 trong Hình học

Cách của thầy cũng giống hệt cách của Tuấn là tính hết ra và kết quả cuối cùng đúng như thế kia. Tuy nhiên Phước có cách giải thuần túy hình khá đẹp các bạn hãy cứ thử sức tiếp. Tuấn đề nghị một bài tiếp đi em!




#670121 VMF's Marathon Hình học Olympic

Đã gửi bởi quanghung86 on 27-01-2017 - 10:50 trong Hình học

Cám ơn em đã đăng lời giải, bài toán 150 có thể tham khảo lời giải dùng tỷ số kép ở đây.

 

Bài toán 151. Cho tam giác $ABC$ với đường cao $AD$. $E,F$ là hình chiếu của $D$ lên $CA,AB$. $EF$ cắt $BC$ tại $T$. $BE$ cắt $CF$ tại $S$. $K$ là tâm ngoại tiếp tam giác $SEF$. Một đường thẳng bất kỳ đi qua $T$ cắt $(ABC)$ tại $M,N$. Chứng minh rằng đường tròn có tâm trên $AK$ và đi qua $M,N$ thì tiếp xúc $(K)$.




#670526 VMF's Marathon Hình học Olympic

Đã gửi bởi quanghung86 on 31-01-2017 - 02:34 trong Hình học

Figure4315.png

 

Lời giải bài toán 164. Gọi $AD$ là đường kính của $(O)$ ngoại tiếp $ABC$ thì $D,H$ đối xứng qua $K$. Từ đó gọi $P,Q$ là hình chiếu của $I$ lên $CA,AB$ và $R,S$ là hình chiếu của $H$ lên $CA,AB$ thì $AR=AB=c,AS=AC=b,AP=AQ=p-a,AF=p-b, AE=p-c$. Từ đó sử dụng tích vô hướng 2 vector.

 

$$\vec{IH}.\vec{EF}=\vec{IH}(\vec{AF}-\vec{AE})=QS.AF-PR.AE=(p-b)(c-(p-a))-(p-c)(b-(p-a))=0.$$

 

Nhận xét. Bài này có lẽ thu được từ bài quen thuộc từ việc lấy đối xứng trục.

 

Bài toán 165 (Crux). Cho tam giác $ABC$ có $P,Q$ là hai điểm đẳng giác. Giả sử hai trong ba tỷ số $\frac{AP}{AQ},\frac{BP}{BQ},\frac{CP}{CQ}$ bằng nhau. Chứng minh rằng cả ba tỷ số bằng nhau.




#670523 VMF's Marathon Hình học Olympic

Đã gửi bởi quanghung86 on 31-01-2017 - 00:20 trong Hình học

Cám ơn Hiếu, đúng là phần quan trọng nhất của bài toán này là từ bổ đề 2. Tuy nhiên việc phát biểu bài toán là từ một bài Iran trên AoPS, trong đó cũng có lời giải của Bảo. Em hãy đề nghị tiếp một bài toán cho topic được duy trì.




#670156 VMF's Marathon Hình học Olympic

Đã gửi bởi quanghung86 on 27-01-2017 - 17:16 trong Hình học

Bài toán 151 này là mở rộng của mình cho một bài toán của bạn Trịnh Huy Vũ, được post tại đây, xem #11, vẫn còn bài toán 146 chưa có lời giải, nếu tới mai không có ai giải mình sẽ post đáp án. Dark Repulsor em hãy đề nghị một bài toán tiếp.




#636113 VMF's Marathon Hình học Olympic

Đã gửi bởi quanghung86 on 27-05-2016 - 23:10 trong Hình học

Bài toán 10' tương đương với việc cần chứng minh $\lim_{n\to\infty}OG_n=0$ với $G_n$ là trọng tâm tam giác $A_nB_nC_n$ và $O$ là tâm đường tròn ngoại tiếp tam giác $A_0B_0C_0$.
 
Bài toán này lần đầu tiên xuất hiện ở đây https://cms.math.ca/...x_v9n05_May.pdf bài toán 844 trang 143
 
Lời giải có ở đây https://cms.math.ca/..._v10n08_Oct.pdf trang 264
 
Chú ý bài toán cho không gian được giải bởi thầy Nguyễn Minh Hà với kỹ thuật đó cũng giải được cho phẳng, bài báo của thầy Hà ở đây https://cms.math.ca/crux/v36/n8/
 
Bài toán chúng ta đóng góp ở đây không nhất thiết mới, đưa lên để thảo luận, tìm lời giải mới. Nếu nguồn gốc ở đâu thì ghi rõ ở đó, còn nếu không nhớ hoàn toàn có thể ghi "sưu tầm", ai nhớ nguồn gốc thì dẫn lại là chuyện bình thường, không có vấn đề gì cả.
 
$\boxed{\text{Bài toán 11.}}$ Cho tam giác $ABC$ nội tiếp đường tròn $(O)$ và $P$ di chuyển trên cung $BC$ không chứa $A$. Đối xứng của $PA$ qua $PB,PC$ lần lượt cắt $AB,AC$ tại $F,E$. Một đường thẳng vuông góc với $PA$ tại một điểm chia $PA$ tỷ số cố định cắt tiếp tuyến tại $A$ của $(AEF)$ tại $Q$. Chứng minh rằng $Q$ luôn thuộc đường thẳng cố định khi $P$ thay đổi.

Nguồn gốc: Sáng tác.



#640183 VMF's Marathon Hình học Olympic

Đã gửi bởi quanghung86 on 14-06-2016 - 02:06 trong Hình học

Topic hơi gián đoạn do bài toán 35 của Dương khá khó, tạm gác lại bài đó, mình xin đề xuất một bài tiếp như sau

 

$\boxed{\text{Bài toán 36.}}$ Cho tam giác $ABC$ cân tại $A$ nội tiếp đường tròn $(O)$ với $AD$ là đường kính của $(O)$. Các điểm $E,F$ lần lượt thuộc $DC,DB$. $G$ thuộc $EF$ sao cho $\frac{GF}{GE}=\frac{FB}{CE}$. Chứng minh rằng $CG$ và $AF$ cắt nhau trên $(O)$.

 

Khi đề xuất bài mình nghĩ nên có một số thứ tự các tiêu chí ưu tiên như sau

 

1) Đúng

2) Hay, đẹp

3) Mới

4) Sát với thi Olympic

5) Kiến thức đơn giản

6) Đề bài ngắn gọn, dễ hiểu

7) Nhiều hướng tiếp cận, tổng quát.




#646777 VMF's Marathon Hình học Olympic

Đã gửi bởi quanghung86 on 27-07-2016 - 19:35 trong Hình học

Hệ quả đẹp của bài toán 86, hai tam giác $IAC,IBD$ có chung đường đối trung.




#644138 VMF's Marathon Hình học Olympic

Đã gửi bởi quanghung86 on 08-07-2016 - 18:50 trong Hình học

Dương đồng ý, mình xin đề nghị một bài khác

 

$\boxed{\text{Bài toán 73.}}$ Cho tứ giác $ABCD$ nội tiếp đường tròn $(O)$. $AD$ cắt $BC$ tại $E$. $AC$ cắt $BD$ tại $G$. $H$ là hình chiếu của $O$ lên $EG$. Trung trực $EH$ cắt $BC,AD$ tại $M,N$. Đường tròn $(HMB)$ và $(HNA)$ cắt nhau tại $K$ khác $H$. Chứng minh rằng $\triangle KBE\sim\triangle KEA$.

 

Figure3937.png

Cám ơn Quân(halloffame) đã nhắc mình chỉnh lại đề. Tiện thể mình thêm hình vẽ.




#644082 VMF's Marathon Hình học Olympic

Đã gửi bởi quanghung86 on 08-07-2016 - 10:17 trong Hình học

$\boxed{\text{Bài toán 71.}}$ Chứng minh rằng đường thẳng Simson của một điểm nằm trên đường tròn ngoại tiếp một tam giác chia đôi đoạn thẳng nối điểm đó và cực trực giao của đường thẳng Simson này với tam giác đã cho.




#643321 VMF's Marathon Hình học Olympic

Đã gửi bởi quanghung86 on 02-07-2016 - 19:27 trong Hình học

Nguồn gốc của bài toán 57 là bài số 14 trong này http://geometry.ru/o...13/zaochn-e.pdf

 

Đáp án ở đây http://geometry.ru/o...13/zaochsol.pdf

 

Bài này là bài toán hay có nhiều phát triển và ứng dụng.




#649811 VMF's Marathon Hình học Olympic

Đã gửi bởi quanghung86 on 15-08-2016 - 21:59 trong Hình học

Chuẩn rồi Bảo, cái điểm đồng quy của AX,BY,CZ đó gọi là điểm Prasolov đó. Điểm Prasolov nằm trên đường nối tâm Euler và điểm Lemoine :)!




#666885 VMF's Marathon Hình học Olympic

Đã gửi bởi quanghung86 on 03-01-2017 - 23:30 trong Hình học

Đáp án bài toán 100.

 

Trong suốt bài toán này nếu $X$ là một điểm nằm trên đường tròn ngoại tiếp tam giác $ABC$ thì ta ký hiệu $s_X$ chỉ đường thẳng Simson của $X$ ứng với tam giác $ABC$. Ta sử dụng các bổ đề sau

 

Bổ đề 1. Cho tam giác $ABC$ nội tiếp đường tròn $(O)$ và $M,N$ thuộc $(O)$ thì $(s_M,s_N)=\frac{1}{2}(\overrightarrow{ON},\overrightarrow{OM})(\bmod\pi)$.
 

Bổ đề 2. Cho tam giác $ABC$ nội tiếp đường tròn $(O)$ và trực tâm $H$ và $P$ là một điểm bất kỳ trên $(O)$ thì $s_P$ đi qua trung điểm $PH$.
 

Hai bổ đề là các bài toán cơ bản xin không nêu cách chứng minh.

 

Bổ đề 3. Cho tam giác $ABC$ nội tiếp đường tròn $(O)$ và $M,N$ là hai điểm thuộc $(O)$. $P$ là một điểm bất kỳ trên đường tròn đường kính $MN$. Gọi $m,n$ là hai đường thẳng lần lượt qua $M,N$ theo thứ tự song song với $s_M,s_N$. Khi đó thì đối xứng của $m,n$ lần lượt qua $PM,PN$ cắt nhau trên $(O)$.

 

Figure2231.png

 

Chứng minh. Bổ đề được chứng minh đơn giản bằng cộng góc. Thật vậy, gọi $m'$ và $n'$ lần lượt là đối xứng của $m,n$ qua $PM,PN$. Ta có biến đổi góc

 

$(m',n')=(m',PM)+(PM,PN)+(PN,n')(\bmod\pi)$

 

$=(PM,m)+(PN,PM)+(n,PN)(\bmod\pi)$ (Do $PM\perp PN$ nên $(PM,PN)=\frac{\pi}{2}=-\frac{\pi}{2}=(PN,PM)(\bmod\pi)$)

 

$=(n,m)(\bmod\pi)$

 

$=\frac{1}{2}(\overrightarrow{OM},\overrightarrow{ON})(\bmod\pi)$.

 

Ta chú ý $m'$ và $n'$ đi qua $M,N$ nên từ đó $m'$ và $n'$ cắt nhau trên $(O)$. Ta có điều phải chứng minh.

 

Giải bài toán.

 

Figure2232.png

 

Gọi $K',L'$ là ảnh của $K,L$ qua phép vị tự tâm $H$ tỷ số $2$. Vì $S$ cũng là ảnh của $T$ qua phép vị tự tâm $H$ tỷ số $2$ nên ta chỉ cần chứng minh $S$ là trung điểm của $K'L'$ thì bài toán được giải quyết. Thật vậy, theo bổ đề 2 dễ thấy phép vị tự tâm $H$ biết $s_M,s_N$ thành các đường thẳng $K'M,K'N$. Từ đó ta có $(K'M,K'N)=(s_M,s_N)=\frac{1}{2}(\overrightarrow{ON},\overrightarrow{OM})(\bmod\pi)$.

 

Vậy nếu gọi $R_1$ là đối xứng của $K'$ qua $MN$ thì $(R_1M,R_1N)=-(K'M,K'N)=\frac{1}{2}(\overrightarrow{OM},\overrightarrow{ON})(\bmod\pi)$ vậy suy ra $R_1$ thuộc $(O)$.

 

Tương tự nếu $R_2$ đối xứng $L'$ qua $PQ$ thì $R_2$ thuộc $(O)$.

 

Mặt khác từ việc lấy đối xứng ta có $R_1N,R_2P$ lần lượt là đối xứng của $NK',PL'$ theo thứ tự qua $SN,SP$. $S$ lại nằm trên đường tròn đường kính $NP$ vậy theo bổ đề 3 thì hai đường thẳng này phải có chung một điểm thuộc $(O)$. Vậy từ đó $R_1\equiv R_2\equiv R\in (O)$. Từ đó theo tính chất đối xứng trục, hai trục $MN,PQ$ vuông góc do đó $L'=\mathcal{R}_{PQ}(R)=\mathcal{R}_{PQ}[\mathcal{R}_{MN}(K')]=\mathcal{R}_{PQ}\circ \mathcal{R}_{MN}(K')=\mathcal{S}_{S}(K')$ vậy $K'$ và $L'$ đối xứng nhau qua $S$. Từ các nhận xét ban đầu ta thu được điều phải chứng minh.




#640216 VMF's Marathon Hình học Olympic

Đã gửi bởi quanghung86 on 14-06-2016 - 09:58 trong Hình học

Cám ơn Bảo, lời giải của thầy
 

Figure2329.png

 

Gọi đường tròn ngoại tiếp tam giác $DEF$ cắt $(O)$ tại $S$ khác $D$. $EF$ cắt $BC$ tại $T$. Các tam giác $\triangle SFB\sim\triangle SEC$ do đó $\frac{SF}{SE}=\frac{BF}{CE}=\frac{GF}{GE}=\frac{TF}{TE}$ đẳng thức cuối có do định lý Menelaus cho tam giác $DEF$ với $T,B,C$ thẳng hàng. Từ đó ta có hàng $(EF,GT)=-1$. Gọi $M$ là trung điểm $BC$, $CF,CG$ cắt $DM$ tại $P,Q$. Chiếu xuyên tâm $C$ suy ra hàng $(DQ,PM)=C(DQ,PM)=(EF,PM)=-1$ hay $(MP,DQ)=-1$ suy ra $(MD,PQ)=1-(MP,DQ)=2$ vậy $\frac{PM}{PD}=2\frac{QM}{QD}\quad (1)$. 
 
Áp dụng định lý Menelaus cho tam giác $DBM$ với $C,Q,F$ thẳng hàng suy ra $\frac{FB}{FD}.\frac{QD}{QM}.\frac{CM}{CB}=1$ suy ra $2\frac{QM}{QD}=\frac{FB}{FD}\quad (2)$.
 
Từ (1),(2) suy ra $\frac{PM}{PD}=\frac{FB}{FD}$, vậy $PF\parallel BC$. Gọi $CP$ giao cắt $(O)$ tại $H$ khác $C$. Suy ra $\angle FPH=\angle BCH=\angle BDH$ suy ra từ giác $HFPD$ nội tiếp. Suy ra $\angle PHF=\angle PDF=\angle ADB=\angle CHA$ vậy $A,H,F$ thẳng hàng. Ta có điều phải chứng minh.



#636016 VMF's Marathon Hình học Olympic

Đã gửi bởi quanghung86 on 27-05-2016 - 20:20 trong Hình học

Nguồn gốc bài 9 http://www.artofprob...unity/c6h486466

Nguồn gốc bài 10 http://www.artofprob...unity/c6h470341




#666953 VMF's Marathon Hình học Olympic

Đã gửi bởi quanghung86 on 04-01-2017 - 20:31 trong Hình học

Lời giải sau do bạn Nguyễn Lê Phước gửi tới tác giả.

 

Figure4051.png

 

Lời giải bài toán 101. Gọi $NI$ cắt $HK$ tại $S$, $MI$ cắt $HL$ tại $T$ thì $S,T$ là tâm bàng tiếp tam giác $ANH,AMH$. Từ đó $\angle ASE=45^\circ=\angle AHL$ và $\angle AES=90^\circ+\angle ANE=\angle ALH$. Từ đó hai tam giác $ASE$ và $ALH$ đồng dạng g.g. Lại có $\angle ATH=90^\circ-\frac{\angle AMN}{2}=180^\circ-\angle AIN=\angle AIS$. Từ đó hai tam giác $ATH$ và $AIS$ đồng dạng g.g. Vậy $\frac{ES}{EI}=\frac{LH}{LT}$. Tương tự $\frac{FT}{FI}=\frac{KH}{KS}$. Từ đó $\frac{FI}{FT}.\frac{LT}{LH}=\frac{EI}{ES}.\frac{KS}{KH}$. Dùng định lý Menelaus dễ thấy $FL,ES$ cùng đi qua một điểm trên $IH$.

 

Lời giải khác của bạn Nguyễn Đức Bảo có thể xem tại http://artofproblems...1308940p7009306, bài toán này nằm trong chuỗi bài mở rộng bài toán IMO 2009 của tác giả http://analgeomatica...-2009-ngay.html

 

Bài toán 102 (Tập huấn đội IMO 2016)Cho tam giác $ABC$ nội tiếp đường tròn $(O)$ cố định. $B$, $C$ cố định, $A$ di chuyển trên $(O)$. $I$ là tâm đường tròn nội tiếp tam giác $ABC$. $K$, $L$ theo thứ tự là trực tâm các tam giác $IAB$, $IAC$. $P$ đối xứng với $O$ qua trung điểm $KL$. Chứng minh rằng $AP$ đi qua một điểm cố định khi $A$ thay đổi.




#642709 VMF's Marathon Hình học Olympic

Đã gửi bởi quanghung86 on 29-06-2016 - 07:37 trong Hình học

Bài này phải dùng nhiều kỹ thuật, mặt khác vì nó là bài của mình tạo ra mình cũng không muốn đưa vào topic, vậy ta tạm gác lại, mình đề xuất một bài toán khác thay thế. 

 

$\boxed{\text{Bài toán 49.}}$ Cho tam giác $ABC$ và $P$ là điểm bất kỳ trong tam giác. Đường tròn $(K),(L)$ lần lượt ngoại tiếp các tam giác $PAC,PAB$ lần lượt cắt $BC$ tại $F,E$ khác $C,B$. $M$ là tâm ngoại tiếp tam giác $AEF$. $PA$ cắt $BC$ tại $D$. Trung trực $AD$ cắt đường thẳng qua $D$ vuông góc $BC$ tại $N$. $MN$ cắt $KL$ tại $Q$. Chứng minh rằng $Q$ nằm trên trung trực $BC$.

 

Mình nghĩ rằng nên có thêm một quy tắc, là những ai đề xuất bài vào topic này cần có đáp án, nếu bài không có đáp án chỉ nên thảo luận bên ngoài thôi.